15
$\开始组$

素数定理的某些版本提供了算术级数中素数的渐近行为$qn+a美元$具有$(q,a)=1$,$n\ge 1美元$.我想知道是否有切比雪夫类型的参数(使用二项式系数或其变体)确定了至少$$c x/\log x$$这种算术级数中的素数x美元$对于一些甚至很小的$c>0$和的一些值$a,q>1美元$例如,形式的素数有这样的论证吗4000美元\pm 1$? 这主要是由我即将教授的一门数论课程引发的好奇。

$\端组$
  • 2
    $\开始组$ 比较$n!$的因式分解乍一看,$\prod_{k^2+l^2<n}(k^2+1^2)$可能会给出一些非平凡的东西。 $\端组$ 评论 2019年8月28日10:31
  • $\开始组$ 在本文中mathscinet.ams.org/mathscinet-gettem?mr=1220462Granville探索了在不必处理类数或Siegel零理论的情况下,PNT的Erdos-Selberg初等证明可以推进多远。粗略地说,只要$a$不是二次剩余(或者更准确地说,位于每个索引之外$({\bf-Z}/q{\bf Z})^\次$的两个乘法子群),他就可以得到原始剩余类$a\hbox{mod}q$中素数的预期下界。因此,例如$4k-1$可以工作,但$4k+1$不行!但这些参数不是切比雪夫类型的。 $\端组$
    – 陶哲轩
    评论 2019年8月28日16:53
  • $\开始组$ 对不起,也许我错过了这个,但为什么我们要排除塞尔伯格的初等证明呢?参见他的论文“算术级数素数定理的初等证明”和“关于算术级数中素数的Dirichlet定理的初等等证明”。可能它们不符合“切比雪夫式”的限制,但无论如何,它们值得一提。 $\端组$
    – 阿普吉
    评论 2019年9月6日15:58

3个答案

重置为默认值
16
$\开始组$

在第9节

哈罗德·戴蒙德。,素数分布研究中的基本方法,公牛。美国数学。Soc.,新系列。7, 553-589 (1982).ZBL0505.10021号.

来自的论点

哈罗德·戴蒙德(Harold G.Diamond)。;埃尔德斯,保罗,关于锐利的初等素数估计,Enseign。数学。,二、。Sér。26, 313-321 (1980).ZBL0453.10007号.

绘制了草图,其中显示了对于每个$\varepsilon>0$有一个切比雪夫式的论点表明

$$(1-\varepsilon)x\leq\psi(x)\leq(1+\varepsilon)x$$

对于所有足够大的x美元$。这个想法是比较$\Lambda(n)=\mu*\log(n)$使用截断版本$\Lambda_T(n)=\mu_T*\log(n$哪里$\mu_T$$\亩$截断为较大的间隔$1,T]美元$(修改时间:T美元$确保正常化$\sum_n\mu_T(n)/n=0$). 基本方法可以表明$|\sum_{n\leq x}\Lambda_T(n)-m_1(T)x |\leq o(x)$对于一个明确可计算的量$1(吨)$(事实上$m_1(T)=\sum_{n<T}\log\frac{T}{n}\frac}\mu(n)}{n{$)可以很容易地证明收敛为$T\到\输入$在适当的平均意义上。特别地$|\sum_{n\leq-x}\Lambda_T(n)-x|\leq|m_1(T)-1|x+o(x)$。Dirichlet双曲线方法也可用于生成上限$|\sum_{n\leq x}\Lambda_T(n)-\psi(x)|\leq\delta_T x+o(x)$对于一些明确可计算的量$\增量_$取决于T美元$; 在PNT的帮助下(在表格中$\sum_{n\leqx}\mu(n)=o(x)$)可以证明这一点$\delta_T\到0$作为$T\到\输入$,这足以结束这一主张。有趣的是,这给出了PNT假设PNT的循环证明;然而,如果人们只想使PNT达到给定的错误$\varepsilon美元$那么这个参数不需要PNT,只需找到一个具体的值T美元$其上限$|m_1(T)-1|,\delta_T$在上述不等式中(可通过切比雪夫类型方法显式计算)足够小。

看起来很可能有人可以通过字符来扭曲这个论点,并得出以下结论:$\varepsilon>0$和本原剩余类$a\hbox{mod}q$有一个切比雪夫式的论点表明

$$(1-\varepsilon)\压裂{x}{\phi(q)}\leq\psi(x,q,a)\leq(1+\varepsilon)$$

对于所有足够大的x美元$,在算术级数中再次需要素数定理$\sum_{n\leq x}\mu(n)\chi(n)=o(x)$)以保证误差的基本上界确实很小。虽然对于特定的残留物类别,这似乎是合理的$1\hbox{mod}4美元$$-1\hbox{mod}4$实际上可以生成显式的数值选择T美元$参数遵循所有必需的属性,特别是产生切比雪夫类型的非平凡下界。

$\端组$
6
  • $\开始组$ 非常感谢!这看起来很有趣。我将仔细研究这些论点,看看它是否适用于算术级数。 $\端组$ 评论 2019年8月28日19:36
  • 1
    $\开始组$ 如果你有机会的话,我很想看清楚这一点。我没有按照草图做,但我可能只是缺少了一些基本的东西。 $\端组$ 评论 2019年9月1日0:48
  • $\开始组$ 为了表明我并不是完全懒惰,下面是我如何理解切比雪夫式参数的:标识$\sum_{d|n}\Lambda(d)=\logn$意味着$\sum_d\LambdaN/d\rfloor=\sum_}n\leqN}\logn=n\logN-n+O(\logN)$。然后,我们对不同的$j$取两边的线性组合,其值为$N/j$,从而得到$\sum_d\Lambda(d)\sum_j\mu_T(j)\lfloor N/(jd)\rfloor=c N+O(\log N)$的显式常数$c$。通过仔细选择$\mu_T$,可以安排$\sum_j\mu_T(j)\lfloor x/j\rfloor$接近$1$。 $\端组$ 评论 2019年9月1日1:06
  • $\开始组$ 但是$\sum_{d|n}\chi(d)\Lambda(d)$似乎不易处理,所以我不知道如何在$\sum_d\chi。 $\端组$ 评论 2019年9月1日1:08
  • 1
    $\开始组$ 我们应该把标识$\sum_{d|n}\Lambda(d)=\logn$看作$\sum_{d|n}\Lambeda(d,d)1(n/d)=\Logn$,其中$1$是常量函数。$\chi$对这个恒等式的扭曲是$\sum_{d|n}\chi(d)\Lambda(d)\ chi(n/d)1(n/d)=\chi。函数$\lfloorx\rfloor=\sum_{n<x}1$现在将被求和函数$\sum_}n<x{chi(n)$替换。 $\端组$
    – 陶哲轩
    评论 2019年9月1日1:26
8
$\开始组$

以下可能会让你高兴:我认为有一些基本的证据$$\sum_{p\leqN}\frac{\logp}{p}=\logN+O(1)\\mbox{和}$$ $$\sum_{p\leq N,\p\equiv 1\bmod 4}\frac{\log p}{p}=\frac{1}{2}\log N+O(1)$$这还不足以暗示PNT,但它确实表明“一半的素数是1美元\b月4日$“有一定的权重。我从一个Vesselin Dmitrov的回答,我将尝试添加一些细节。我不知道这个论点是否或在哪里出现在印刷品中。

$$F=\sum_{0<n\leq n}\log n\\mbox{和}\G=\sum_{0<u^2+v^2\leq n{\log(u^2+v^2)$$所以$F(美元)$$\log N$G美元$是的高斯模拟$F(美元)$.

我们可以通过将总和转换为积分来估计每个值。相应的积分为$$\int_{t=0}^N\log-tdt=N\log N-N\\mbox{和}\\int_{r=0}^{\sqrt{N}}2\pi r\log(r^2)dr=\pi N\log-N-\pi N$$转换为积分会导致误差$O(\log N)$在第一种情况下$O(\sqrt{N}\log N)$第二个,但我们只需要领先的条件,所以$$F=N\log N+O(N)\\mbox{和}\G=\pi N\logN+O(N)$$

像往常一样,我们写$\lfloor x \rfloor$对于x美元$四舍五入到最接近的整数,我们认为它是区间中的整数数$(0,x)$受此启发,让$\lfloor x \rfloor_2$是晶格点的数量$(u,v)$具有$0<u^2+v^2\leq x$显然,$\lfloor x\rfloor_2=\pi x+O(\sqrt{x})$.

通常的论点告诉我们$$F=\sum_p\sum_{k\geq1}\left\lfloor\frac{N}{p^k}\right\rfloor\log p=\sum__{p\leqN}\frac}{p}\log p+O(N)$$所以$$\sum_{p\leq N}\frac{N}{p}\log p+O(N)=N\log N+O(N)$$并除以N美元$,声明如下。

模拟常用参数,但在中使用唯一因子分解$\mathbb{Z}[i]$,我们得到$$G=\sum_{\pi}\sum_{k\geq 1}\left\lfloor\frac{N}{N(\pi)^k}\right\rfloor_2\log N(\π)$$其中和在高斯素数上运行$\pi美元$。我们可以将其重写为$$G=2\sum_{p\equiv1\bmod4}\sum_{k\geq1}\left\lfloor2\frac{N}{p^k}\right\rfloor_2\log p+\sum_}p\equav3\bmod4{sum_k\geq 1}\leaft\lffloor\fracc{N{p^{2k}}\rift\rfloroor_2\log p^2+\sum_{k\ageq1}\left\ lfloor\frac{N}{2^k}\right\rfloor_2\log 2$$后面的总和可以忽略不计,并且接近$\lfloor x \rfloor_2$通过$\πx$给予$$G=2\sum_{p\equiv1\bmod4,\p\leq N}\frac{\pi N}{p}\log p+O(N)$$比较我们的公式G美元$并用美元\pi N$给予$$2\sum_{p\equiv 1\bmod 4,\p\leq N}\frac{\log p}{p}=\log N+O(1)$$正如承诺的那样。

如果我稍后有机会,我可能会评论除此之外的其他模$4$.

$\端组$
6
  • $\开始组$ 谢谢,这很好,和预期的一样。所以现在使用标准的切比雪夫技巧,人们甚至可以在不加权的情况下,得到算术级数中素数的正确阶下界。 $\端组$ 评论 2019年8月30日9:43
  • $\开始组$ 我认为这行不通?选择整数$b>1$。设$Q$是一组以$b$为基数的素数为偶数的素数。然后我得到q中的$\sum_{q\,\q\leqN}\tfrac{\logq}{q}=\tfrac}{1}{2}\logN+O(N)$。但是,q中的$\sum_{q\leqN}1$可以小到$\tfrac{N}{b\logN}$。 $\端组$ 评论 2019年8月30日11:35
  • 2
    $\开始组$ 我试图模仿阿波斯托的ANT中的一个论点。让我尝试在这里重现它。设置$$A(N)=\sum_{p\equiv1\bmod{4},\p\leN}\frac{\logp}{p}.$$您的参数显示$A(N)=\frac{1}{2}\logN+G(N)$,其中$|G(N)|\leM$。这意味着对于$\epsilon>0$和$N\gg_{\epsilon}1$,我们有$$A(N)-A(\epsillon N)=-\frac{1}{2}\log\epsi隆+G(N)-G。 $\端组$ 评论 2019年8月30日12:09
  • $\开始组$ 通过选择适当的$\epsilon$,使上述不等式的右手边为1,我们可以保证$A(N)-A(\epsilonN)\ge 1$与$N\ge\epsilon^{-1}$一样长。现在给出了$N\gg 1$$1\leA(N)-A(\epsilon N)=\sum_{p\equiv1\bmod{4},\\epsilonN<p\leN}\frac{\log p}{p}\le\frac{1}{\epsilen N}\sum_{p\Equiv1\\bmod{4],\p\leN}{\logp}\frac{\logN}{\ebsilon N}od{4},\p\le N\}.$$这行得通吗? $\端组$ 评论 2019年8月30日12:09
  • 4
    $\开始组$ 我发现有三篇论文中出现了这种论点(显然,独立于前一篇)。1.H.Poincare:M.Tchebicheff理论的前体复合体延伸(1892)网站.mathdoc.fr/JMPA/PDF/JMPA_1892_4_8_A2_0.PDF2.A.Selberg:第71-72页,共页算术级数质数定理的初等证明(1950). 3.R.Breusch:4n+1形式素数的渐近公式。(密歇根数学杂志,1964年)项目euclid.org/euclid.mmj/1028999182 $\端组$ 评论 2019年9月1日2:53
4
$\开始组$

这是另一种更像切比雪夫的方法。这可能是特里画的证明,我只是不理解。我认为这应该推广到AP的任何模数,但我会坚持$4$.让美元\chi$是二次字符模$4$为了证明美联社mod 4中的PNT,我们必须证明$$\sum_{n\leq n}\Lambda(n)=n+o(n),\\sum_{n\leq n}\ chi(n)\Lambda(n)=o(n)$$切比雪夫式参数证明了形式的边界$a N<\sum_{N\leq N}\Lambda(N)<b N$; 我将类推证明形式的边界$\left|\sum_{n\leq n}\chi(n)\Lambda(n)\ right|<cN$。只有当我设法获得$c<1$.

对于任何正实数x美元$,定义$$\langle x\langle=\sum_{1\leq k\leq x}\chi(k)$$所以$\langle x等级$$1$如果$\lfloor x \rfloor\equiv 1,2\bmod 4美元$$0$如果$\lfloor x \rfloor\equiv 3,4\bmod 4美元$.

放置$$S=\sum_{n\leq n}\chi(n)\log n$$首先要注意$$\左|S\右|\leq\log N$$因为每一项都转换了部分和的符号。

将总和展开为$$S=\sum_{n\leq-n}\sum_}d\chi(d)\chi(n/d)\Lambda(d)=\sum_d\chi!N/d范围。\四元(\ast)$$

现在,$\langle N/d范围$$1$对于$N/3<d\leq N$,是$0$对于$N/5<d\leq N/3$和介于之间$0$$1$为所有人d美元$,所以$$\left|\sum_{N/3<d\leq N}\chi(d)\Lambda(d)\ right|\leq|S|+\sum__{d\leqN/5}\Lambda(d)$$调用切比雪夫边界$\sum_{d\leq M}\Lambda(d)\leq 1110万$,我们有$$\left|\sum_{N/3<d\leq N}\chi(d)\Lambda(d)\ right|\leq\log N+1.11\ times\frac{1}{5}N$$总结以下方程式N美元$,美元N/3$,美元N/9$, ..., 我们得到$$\left|\sum_{d\leqN}\chi(d)\Lambda(d)\ right|\leq\frac{(\logN)^2}{\log3}+1.11\ times\frac{3}{10}N$$0.333美元<1$,我们赢了。

我认为有很大的空间来改进这一论点。如果我们把方程加起来$(\ast)$N美元$美元N/3$,我们得到$$\sum_d\chi(d)\Lambda(d){\大(}\langle N/d\rangle+langle N/(3d)\rangle{\Big)}=O(\log N)$$我们有${\大(}\langle N/d\rangle+langle N/(3d)\rangle{\Big)}=1$对于$N/7<d\leq N$,然后$=0$对于$N/9<数字N/7$,并且总是最多$2$。所以类似的论据给出了$$\left|\sum_{N/7<d\leq N}\chi(d)\Lambda(d)\ right|\leq 1.11\times\frac{2}{9}N+O(\log N)$$从而取代$1.11\次\tfrac{3}{10}$具有1.11美元\次\压裂{16}{63}$归根结底。我认为如果我们采用更聪明的线性组合,这个常数可以被追赶得更低。

更准确地说,我们有$\sum_{t=1}^t(t)\mu(t)\ langle x/t范围=1$对于$x\geq吨$因此,对于固定T美元$,我们有$$\sum_{t=1}^t\mu(t)\chi(t!N/(td)范围=$$ $$\sum_{N/T<d\leq N}\chi!N/(td)\rangle\right)=O(\log N)$$如果我们能得到带括号的数量0美元(吨)$,独立于N美元$,我们会赢的。

$\端组$
2
  • $\开始组$ 谢谢你的详细回答。为了澄清一个小问题:要获胜(仅使用切比雪夫的界限),需要系数0.333小于切比雪夫下界(约0.9),而不是1,对吗?当然,这也适用。 $\端组$ 评论 2019年9月1日10:55
  • $\开始组$ 是的,没错。 $\端组$ 评论 2019年9月1日11:53

你的答案

单击“发布您的答案”,表示您同意我们的服务条款并确认您已阅读我们的隐私政策.

不是你想要的答案吗?浏览标记的其他问题问你自己的问题.